Chest/Abdomen/Trunk/Rec/Congenital Flashcards
Identify true versus false ribs.
A) 1 thru 7 are true ribs and 8-12 are false ribs
B) 1 thru 7 are false ribs and 8-12 are true ribs
C) 1-10 are true ribs and 11-12 are false ribs
D) none of the above
Correct answer is option a
There are a total of twelve paired ribs. The first thru the seventh ribs connect directly to the sternum via the costal cartilage and are referred to as “true ribs”. The eight thru twelfth ribs do not connect directly to the sternum and are referred to as “false ribs”. The eighth thru tenth ribs do have cartilaginous attachments to the ribs above them but do not connect to the sternum; The eleventh and twelfth ribs do not have any attachments with other adjacent ribs and are referred to as “floating ribs”. 
A 24-year-old woman is scheduled to undergo correction of the defect shown in the photographs. Physical examination shows an elevated inframammary fold and herniation of breast tissue through the areolar complex. Which of the following is the most appropriate operative plan?
A) Latissimus dorsi musculocutaneous flaps and placement of silicone gel prostheses
B) Subfascial placement of silicone gel prostheses with mastopexy
C) Subglandular placement of silicone gel prostheses
D) Submuscular placement of silicone gel prostheses
E) Submuscular placement of silicone gel prostheses with mastopexy and scoring of the gland
The correct response is Option E.
Tuberous breast deformity is a rare congenital condition that results in aberrant breast shape due to a constricting ring at the breast base. Abnormal development results in breast tissue deficiency, herniation of breast tissue into the nipple-areola complex, areolar enlargement, and breast asymmetry.
Although latissimus flaps could be used for severe primary cases or reoperative secondary cases, such an aggressive intervention would not be warranted in the patient described. Surgical correction is challenging, but it can be achieved in a single-stage operation. This procedure should include submuscular placement of silicone or saline prostheses, mastopexy with areolar reduction, and scoring of the gland to relieve the constricting tissue. Postoperative results are depicted in the photograph shown.
A 46-year-old woman undergoes breast reconstruction with a pedicled transverse rectus abdominis musculocutaneous (TRAM) flap. Weight is 185 lb (84 kg); BMI is 32 kg/m2. The donor site is reconstructed with mesh. Six months postoperatively, the patient develops a bulge at the donor site. Which of the following best represents the deepest layer of the anterior rectus sheath when it is harvested caudal to the arcuate line?
A) External oblique aponeurosis
B) Internal oblique aponeurosis
C) Rectus abdominis fascia
D) Transversalis fascia
E) Transversus abdominis aponeurosis
The correct response is Option E.
The rectus sheath is the semifibrous compartment that encompasses the rectus abdominis muscle and consists of an anterior and posterior sheath created by the aponeurotic extensions of the external oblique, internal oblique, and transversus abdominis muscles.
Proximal to the arcuate line, which is located approximately at the level of the anterior superior iliac spine, the anterior rectus sheath comprises the aponeuroses of the external oblique and the anterior leaf of the internal oblique. The internal oblique has two leaves, the deeper of which contributes to the posterior rectus sheath along with the transversus abdominis and transversalis fascia.
At the level of the arcuate line, the posterior leaf of the internal oblique aponeurosis and the transversus abdominis aponeurosis travel superficially to the rectus abdominis, making the posterior sheath very weak. When the anterior rectus sheath is defective at this level, the likelihood for bulging or hernia is high.
The level of the arcuate line is not always clear from topographical landmarks, which can make the planning of a transverse rectus abdominis musculocutaneous (TRAM) flap more difficult and increase the risk for a hernia complication.
Musculoskeletal system develops from the ectodermal neural crest and the somatic mesoderm. Further differentiation yields a dorsolateral subpopulation of somatic cells called the ___________, and a ventromedial subpopulation of cells known as the ___________.
A) blastomere and zygote
B) dermomyotome and sclerotome
C) sclerotome and dermomyotome
Correct answer is option B.
The musculoskeletal system develops from the ectodermal neural crest and the paraxial and lateral plate (somatic layer) mesoderm. Approximately 40 segmental tissue blocks alongside the neural crest caudal to the head region, known as somites, differentiate into 2 parts. The dorsolateral subpopulation of somatic cells is called the dermomyotome, and the ventromedial subpopulation of cells is known as the sclerotome. The dermomyotome eventually forms the musculature of the trunk, whereas the sclerotome develops into the skeletal framework.
Which of the following best represents the likelihood of malignancy in adolescents undergoing subcutaneous mastectomy for gynecomastia?
A ) 1%
B ) 5%
C ) 10%
D ) 15%
The correct response is Option A.
A literature search yielded over 2000 articles in total; however, only 36 articles have discussed cases of adolescent gynecomastia and the associated pathologic results, resulting in data for 615 individuals. Of these 615 individuals, there have been six cases of cancer and five cases of atypical ductal hyperplasia associated with adolescent gynecomastia. Specific patient information was only available for seven of the individuals (six breast cancer and one atypical ductal hyperplasia), which revealed that the average age of patients involved was age 17.4 years (range, age 16 to 20 years), 43% of cases had symptoms of unilateral gynecomastia, and an abnormal physical examination was present in only one case. In the cases with histologic characterization, five cases were ductal carcinoma in situ with low to intermediate grades, and the other case had been diagnosed as invasive carcinoma of the secretory carcinoma type. Sixty percent of the cases of ductal carcinoma in situ had associated atypical ductal hyperplasia as well.
A 56-year-old man has a deep soft-tissue defect of the posterior neck with exposure of the vertebral bone after undergoing excision of a malignant tumor. Which of the following would preclude the use of a trapezius flap for coverage of the defect?
A) Atherosclerotic occlusion of the occipital arteries.
B) Atherosclerotic occlusion of the vertebral arteries.
C) Prior endovascular placement of an ipsilateral internal carotid artery vascular stent
D) Prior ipsilateral radical neck dissection
dividing the transverse cervical artery.
E) Prior ligation of the ipsilateral circumflex scapular vessels.
Correct answer is option D.
The transverse cervical artery, which provides the primary vascular supply to the trapezius flap, is typically divided during an ipsilateral radical neck dissection. Therefore, the trapezius flap cannot be used for coverage of a defect in a patient who has undergone an ipsilateral radical neck dissection because its primarily vascular supply is presumed to be ligated. The anterior branch then courses toward the shoulder, while the posterior branch courses beneath the central portion of the trapezius along its main axis, continuing to supply blood to this flat, triangularly shaped type II muscle. The occipital artery is a secondary source of vascularity for the trapezius muscle flap. However, in looking at the specific location and depth of this patient’s defect, it appears that the portion of the muscle supplied by the occipital artery has been resected. The medial edge of the trapezius flap also receives some blood from posterior thoracic intercostal perforators along the medial edge of the flap. Because carotid endarterectomy is performed at or close to the bifurcation of the common carotid artery, it does not disrupt the thyrocervical trunk. The circumflex scapular vessels and vertebral arteries are not involved in supplying vascularity to the trapezius flap.
A 46-year-old man presents with a midline 18-cm-wide ventral hernia 1 year after undergoing midline exploratory laparotomy for a bowel resection and right end ileostomy. Medical history includes significant weight loss through diet and exercise. His weight has been stable for 2 years. BMI is 29 kg/m2. He undergoes bilateral component separation with biologic mesh bridged between the rectus muscles and concomitant panniculectomy. Which of the following clinical characteristics will most likely increase the likelihood of hernia recurrence?
A) BMI greater than 24.9 kg/m2
B) Bridged biologic mesh hernia closure
C) Concomitant panniculectomy
D) Presence of an end ileostomy
E) Prior abdominal surgery
The correct response is Option B.
The patient presents after significant weight loss with a wide midline ventral hernia, right end ileostomy through his rectus muscle, and an abdominal pannus. Given the 18-cm waist of the hernia defect, he is being counseled that only a bridged repair with a biologic mesh will be possible rather than total muscular coverage for the midline defect. Hernia recurrence is a major problem for patients and can be associated with specific characteristics. When the technique of bilateral component separation and inlay biologic mesh repair is being performed, the most important predictor of recurrence is whether the rectus muscle and fascia will be able to be closed at midline, creating a total submuscular repair, or whether the mesh will be bridged. A bridged repair is associated with a 33% chance of recurrence at 3 years compared to 6.2% for total muscle coverage with fascial closure at midline.
With a BMI of 29 kg/m2, the patient remains overweight despite his prior stable weight loss. Surgical site occurrences are increased in the overweight patient with a 26.4% incidence versus 14.9% in patients with BMI less than 24.9 kg/m2. Similarly, skin dehiscence is significantly increased in the overweight patient (19.3% versus 7.2%), while hernia recurrence rates are not statistically significant (11.4% versus 7.7%). Concomitant panniculectomy was associated with an increase in surgical site occurrences and skin dehiscence, but hernia recurrence rates were not affected.
Similarly, patients with existing ileostomies or stomas complicated by parastomal hernias do have a significantly increased surgical site occurrence rate (34.1% with parastomal and midline hernia versus 18.7% with midline hernia only) but hernia recurrence rates are not affected. Prior abdominal surgery will be in the clinical history of all incisional hernia patients.
Where are the nerves of the anterior abdominal wall found?
A) between the external and internal oblique muscles
B) under the fascia
C) deep to the internal oblique muscles
D) In the rectus muscles
Correct answer is option C.
The nerves of the anterior abdominal wall lie deep to internal oblique muscle which is distinct from the avascular plane between the external and internal oblique muscles utilized for the components separation 
Which of the following best describes the benefit of using acellular human dermal matrix compared with alloplastic mesh in complex abdominal wall reconstruction?
A) Adhesion potential
B) Cost effectiveness
C) Definitive scar formation
D) Rapid absorption
E) Resistance to infection
Correct answer is option E.
Acellular human dermal matrix has emerged as a versatile material for abdominal wall reconstruction. Because of retained vascular channels, this material revascularizes rapidly and resists infection. In addition, the matrix serves as a soft-tissue scaffold, retaining its original strength and becoming incorporated with minimal scar or adhesion formation. These materials are more expensive than alloplastic mesh, although a formal cost analysis has not yet been published.
The dominant vascular supply of the rectus abdominis muscle originates from which of the following vessels?
A) Common femoral
B) External iliac
C) Internal iliac
D) Internal mammary
E) Superficial femoral
The correct response is Option B.
Component separation for closure of large abdominal wall defects was first described by Ramirez in 1990. The purpose of the surgery is to achieve abdominal wall closure with well-vascularized, innervated muscle flaps. The primary vascular supply to the rectus muscles are the deep inferior epigastric artery and vein, which arise from the external iliac vessels.
The internal mammary vessels give rise to the superior epigastric arteries and veins, which is a secondary, nondominant vascular supply of the rectus muscles. The femoral vessels give rise to the superficial inferior epigastric artery and vein, which perfuse the skin and subcutaneous fat of the inferior lateral abdomen.
A 1-day-old female newborn is evaluated because of repair of a lumbar myelomeningocele. After dural repair, physical examination shows the spinal cord at the base of the wound with a 4 × 4-cm soft-tissue and skin defect. Which of the following is most appropriate to reconstruct the wound?
A) Gluteal muscle flap and skin advancement flap
B) Paraspinous musculofascial flap and skin advancement flap
C) Skin advancement flap only
D) Split-thickness skin grafting
The correct response is Option B.
The most appropriate method to reconstruct the wound is a local musculofascial flap and skin advancement flap. The major principle of myelomeningocele repair is to obtain a well-vascularized layer of soft-tissue coverage between the dural and skin repairs. The fascia overlying the paraspinous muscles can be turned over as flaps, followed by paraspinous muscle advancement flaps to cover the underlying dural repair. This vascularized soft-tissue layer will minimize the risk of cerebrospinal fluid contact with cutaneous bacteria and subsequent meningitis if either the dural repair or skin repair breaks down.
A split-thickness skin graft over the dura would not adequately protect the spinal cord. Closing the skin directly over the dural repair using skin advancement flaps would place the child at risk for meningitis in the event of a cerebrospinal fluid leak or if wound breakdown occurred along the incision line of the widely undermined skin flaps.
The use of a regional gluteal or latissimus muscle flap to cover the dural repair is unnecessary because local tissue (paraspinous muscles and fascia) is available. Harvesting the gluteal or latissimus muscles also may cause significant donor site morbidity in a child already at risk for ambulatory problems from a neurologic deficit.
A 55-year-old man presents for a large abdominal midline hernia repair. A component separation is planned with a posterior approach and a retrorectus mesh placement. Which of the following layers can be divided to provide further release and preserve the innervation to the rectus muscle?
A) Anterior rectus sheath
B) External oblique
C) Internal oblique
D) Transversalis fascia
E) Transversus abdominis
The correct response is Option E.
In the posterior component separation approach for ventral hernia repair, transversus abdominis release (TAR) can provide further mobility and preserve the innervation to the rectus muscle. The posterior approach reinforces hernia repair with a sublay mesh placed between the rectus muscle and posterior sheath. The Rives-Stoppa approach is associated with a 3 to 6% recurrence rate. To avoid disruption of the segmental nerves to the rectus, classical dissection was limited medial to the linea semilunaris. This, however, limited the space and reserved this technique for small- to medium-sized hernias. To extend this dissection laterally for use in larger defects, either the internal oblique or the transversus abdominis muscle can be divided. Division of the internal oblique divides the nerves to the rectus muscle. Division of the transversus abdominis can preserve these nerves. With this technique, the anterior rectus sheath is preserved as well as the external oblique and transversalis fascia.
A 62-year-old woman presents with a new-onset draining sinus of the left thoracic cage with associated indurated skin. Medical history includes bilateral breast cancer that was managed with bilateral radical mastectomy with radiation therapy 27 years ago. On CT scan, the image (shown) is consistent with osteoradionecrosis. Resection of affected skin, soft tissue, and thoracic cage produces a 35 × 20-cm soft-tissue defect and a skeletal defect spanning five ribs. A photograph of the defect is shown. The thoracic cage is fibrotic and noncompliant because of previous radiation. Which of the following approaches is most appropriate for reconstruction?
A) Free omental flap with skin graft over titanium mesh and reconstruction plates
B) Left latissimus dorsi muscle flap with skin graft over acellular dermal matrix
C) Left rectus abdominis turnover flap with skin graft over methyl methacrylate sandwich
D) Reverse abdominoplasty advancement over ePTFE patch
E) Right pectoralis muscle turnover flap over polypropylene mesh
The correct response is Option B.
The most appropriate option for this patient is a left latissimus dorsi muscle flap with skin graft over acellular dermal matrix, given the alternatives listed. Basic principles of thoracic reconstruction include: debridement of devitalized tissue, removal of foreign bodies, establishment of healthy wound bed, restoration of stability/structure (generally reconstruction of skeleton if more than four ribs or a greater than 5-cm-diameter defect is involved), restoration of normal respiratory mechanics, protection of vital structures/organs, obliteration of dead space, provision of durable coverage, and delivery of an aesthetic result. However, if a patient has been previously irradiated, and therefore loses compliance of the thoracic cage because of radiation-induced fibrosis, skeletal reconstruction may not be mandatory if there is no paradoxical motion of the thoracic cage upon respirations and there is preservation of respiratory efficiency. Such is the case with this patient.
A left rectus turnover flap would not be a good option for two reasons: 1) as can be seen in the image, the left internal mammary artery has been harvested, thereby compromising the superior epigastric vessel on which this flap would be based, and 2) it is insufficient to provide enough soft-tissue coverage of a defect this size. Furthermore, as indicated above, methyl methacrylate would not be mandatory in this patient.
A right pectoralis turnover flap is insufficient to cover a defect this size.
A free omental flap can be used to reconstruct this defect (as can a pedicled omental flap), but again, thoracic skeletal reconstruction would not be mandatory in this previously irradiated patient; furthermore, even if it were, titanium mesh and reconstruction plates would not be utilized.
A reverse abdominoplasty flap (Ryan procedure) would not be able to cover a defect this size.
A 41-year-old man is being evaluated prior to ventral hernia repair. History includes a traumatic abdominal injury with exposed bowel 4 years ago treated with negative pressure dressings and skin grafting. A separation of components technique will be used. The connection between which of the following layers will most likely remain intact during this procedure?
A ) Bowel and fascial edge
B ) External oblique and internal oblique
C ) External oblique and subcutaneous layer
D ) Internal oblique and transversus abdominis
E ) Rectus muscle and posterior rectus sheath
The correct response is Option D.
The principle of component separation is that the layers of the abdominal wall are able to be mobilized to a greater degree when they are separated from one another. In a ventral hernia repair, the bowel is freed from the fascial edges. Large skin flaps are developed that expose the abdominal fascia. The external oblique layer is separated from the internal oblique layer just lateral to the rectus sheath. Incising the posterior rectus sheath can add 1 to 2 cm of additional mobilization if necessary. The internal oblique muscle is usually not separated from the transversus abdominis muscle because it contains the intercostal nerves and blood vessels. This makes dissection difficult, bloody, and heightens the risk of denervating the rectus abdominis muscle.
Which of the following is the most common hyperplastic childhood breast anomaly?
A ) Giant fibroadenoma
B ) Gynecomastia
C ) Juvenile hypertrophy
D ) Polymastia
E ) Polythelia
The correct response is Option B.
Breast anomalies in children are a relatively common finding. A recent study showed that hyperplastic abnormalities were the most common category, with the most common anomalies being gynecomastia, followed by juvenile hypertrophy or hyperplasia. The average age at initial surgery for the group was age 17.4 years; the average number of operations per patient was 1.14. Patients most likely to require revisional surgery were those with giant fibroadenoma.
A 5-year-old boy is evaluated for a cerebrospinal fluid leak and a 3 × 3-cm area of wound dehiscence involving the posterior trunk following tethered cord repair. Which of the following is the most appropriate method for reconstructing the wound?
A) Gluteal muscle flap and skin advancement flap
B) Latissimus muscle flap and skin advancement flap
C) Local fascial flap and skin advancement flap
D) Skin advancement flap only
E) Split-thickness skin graft
The correct response is Option C.
The most appropriate method to reconstruct the wound is a local fascial flap and skin advancement flap. The major principle of tethered cord and myelomeningocele repair is to obtain a well-vascularized layer of soft-tissue coverage between the dural and skin closures. The fascia overlying the paraspinous muscles can be turned over as flaps to cover the underlying dural repair. This vascularized soft-tissue layer will minimize the risk of cerebrospinal fluid leak by reinforcing the dural repair. In addition, the fascial flaps will prevent contact with cutaneous bacteria and subsequent meningitis if either the dural repair or skin repair breaks down.
A split-thickness skin graft over the dura would not adequately protect the spinal cord. Closing the skin directly over the dural repair using skin advancement flaps would place the child at risk for meningitis in the event of a cerebrospinal fluid leak or if wound breakdown occurred along the incision line of the widely undermined skin flaps.
The use of a regional gluteal or latissimus muscle flap to cover the dural repair is unnecessary because local tissue (paraspinous muscle fascia) is available. Harvesting the gluteal or latissimus muscles also may cause significant donor site morbidity in a child already at risk for ambulatory problems from a neurological deficit.
A 33-year-old woman seeks panniculectomy to address intertrigo following a 100-lb weight loss after undergoing bariatric surgery 18 months ago. Her weight has been stable for the past 3 months, and her current BMI is 30 kg/m2. Which of the following aspects of this patient’s history is most likely to interfere with insurance coverage?
A) BMI of 30 kg/m2
B) 3 Months of weight stability
C) 18 Months status postbariatric surgery
D) Primary symptom of intertrigo
E) 100-lb weight loss
The correct response is Option B.
Many patient history factors are important when considering indications for body contouring after weight loss. Surgical indications include symptomatic rashes, large amounts of weight loss, adequate time between bariatric surgery and body contouring surgery, decreased BMI, and a substantial time period of weight stability, longer than 3 months. Additionally, insurance carriers have varying criteria to allow authorization of abdominal contouring procedures, specifically panniculectomy. Many insurance carriers require 6 months of weight stability. Severe intertrigo, 100-lb weight loss, 18 months status post bariatric surgery, and a relatively low BMI would be in keeping with frequently used clinical indications for surgery and insurance coverage criteria.
A 27-year-old woman is scheduled to undergo female-to-male transgender surgery. A fasciocutaneous radial forearm flap is designed to construct the phallus. This procedure includes neurorrhaphy of the lateral antebrachial cutaneous nerve to a terminal branch of which of the following?
A) Genitofemoral nerve
B) Iliohypogastric nerve
C) Ilioinguinal nerve
D) Posterior femoral cutaneous nerve
E) Pudendal nerve
The correct response is Option E.
In women, the pudendal nerve terminates in the dorsal nerve of the clitoris, which is responsible for most erogenous sensation. Neurorrhaphy to this nerve has been shown to result in superior sensory outcomes in phalloplasty. The dorsal nerve of the clitoris is analogous to the dorsal nerve of the penis, which is used as a target for neurorrhaphy during penile reconstruction for amputation or congenital microphallus. Other branches of the pudendal nerve include the inferior rectal and perineal nerves. The ilioinguinal nerve supplies sensation to the skin of the upper medial thigh and to the skin of the mons pubis and labia majora. The genitofemoral nerve also provides sensation to the labia majora and skin of the femoral triangle. The posterior femoral cutaneous nerve innervates the skin of the posterior surface of the thigh as well as part of the perineum. The iliohypogastric nerve innervates the skin above the pubis and in the gluteal region.
A 55-year-old man presents with ventral incisional hernia of the abdomen. BMI is 32 kg/m2. Medical history includes an exploratory laparotomy following a traumatic injury 1 year ago. The abdominal fascia was closed primarily at the end of the procedure. Physical examination shows a fascial deficit 10 cm in width. CT scan demonstrates that rectus muscles are intact bilaterally and 12 cm apart in the periumbilical region. Hernia repair is performed, but the fascia cannot be brought together primarily at the midline. Which of the following operations is most likely to result in the lowest risk for future hernia formation in this patient?
A) Bilateral component separation
B) Bilateral component separation with onlay mesh reconstruction
C) Bilateral component separation with underlay mesh reconstruction
D) Inlay mesh reconstruction
E) Unilateral component separation
The correct response is Option C.
Long-term hernia risk is lowest following primary fascial closure and placement of mesh, either in the retrorectus position or as an underlay. Inlay mesh reconstruction, in which primary fascial closure is not possible and the mesh serves as a bridge, is associated with the highest rates of abdominal hernia formation. Unilateral or bilateral component separation may allow for primary fascial closure, but placement of a mesh augments the repair and reduces hernia recurrence. Regarding onlay mesh placement, in which the mesh is placed superficial to the fascia, hernia recurrence rates have been shown to be higher in obese patients than when the mesh is placed as an underlay.
A 25-year-old man is brought to the emergency department after sustaining injuries during a motorcycle collision. He is alert, cooperative, and vital signs are stable. Examination shows a large skin avulsion with deep lacerations of the left chest and back. Radiograph of the chest shows fractures of multiple ribs; no other abnormalities are noted. During irrigation of the chest wounds, his breathing becomes labored and oxygen saturation and blood pressure decrease precipitously. After stabilizing the airway, which of the following is the most appropriate next step in management?
A ) Administration of a vasopressor
B ) Diagnostic peritoneal lavage
C ) Needle decompression
D ) Placement of central and arterial catheters
E ) Repeat radiograph of the chest
The correct response is Option C.
The most likely diagnosis for a precipitous drop in oxygenation and blood pressure in the setting of a large open wound of the chest is tension pneumothorax. Needle decompression with a large-bore intravenous catheter is performed in the second intercostal space in the midclavicular line. This procedure is followed by tube thoracostomy for stable decompression of the thoracic cavity, thereby allowing lung inflation.
The diagnosis of tension pneumothorax is confirmed by a rush of air escaping the chest on needle decompression. In tension pneumothorax, the negative pressure caused by inspiration draws air through the wound, and the injured tissue acts as a one-way valve. This pressure accumulation in the chest causes collapse of the lungs and impedes cardiovascular filling.
Administration of vasopressors would not be a first-line means of increasing blood pressure in this patient. Decompression of the compressed atria and thoracic vessels along with fluid and blood replacement are the first resuscitative measures required. Vasopressors may play a role for hypotension after appropriate fluid resuscitation fails to normalize pressure.
Diagnostic peritoneal lavage is an emergency department technique for rapid diagnosis of occult abdominal injury. While the patient described is at risk for intra-abdominal trauma, the rapid oxygen desaturation and large chest injuries suggest a thoracic source of injury.
Line placement is needed for ongoing monitoring and care of the critically injured patient described but would not be the initial procedure. Large-bore peripheral intravenous catheters for rapid fluid infusion would be optimal for the early treatment phase of the trauma patient.
A repeat radiograph of the chest may be helpful in establishing a diagnosis, although in the unstable patient with suspected tension pneumothorax, emergent decompression is required. The patient would not likely survive the time interval from ordering a radiograph until the results were made available.
A patient comes into the operating room and undergoes a bilateral latissimus dorsi-gluteus maximus musculocutaneous flap without lateral relaxing incisions. What type of problem is this procedure most likely correcting?
A) Spina bifida occulta
B) Large myelomeningocele
C) Pectus carinatum
D) Intradural spinal cyst
E) Small myelocele
The correct answer is option B.
Large myelomeningoceles may be repaired with a variety of surgical techniques depending on the location, size, and severity of the defect. Among the many methods described in the literature, one could perform a bilateral latissimus dorsi-gluteus maximus musculocutaneous flap for a large myelomeningocele. The other defects described would not require this type of flap repair. Spina bifida occulta is usually benign and asymptomatic. If surgical repair is done, it is for cosmetic purposes such as to remove the pigmented skin over the area. Pectus carinatum would usually be surgically repaired with a modified Ravitch technique. An intradural spinal cyst would involve surgical excision of the cyst. Myeloceles can also vary in size and location and may require different surgical techniques. However, small myeloceles are usually corrected with primary closure and would not require a musculocutaneous flap.
A 3-year-old child with pectus excavatum deformity is evaluated for surgical correction of the chest wall. The child has experienced mild respiratory insufficiency. Which of the following is the optimal timing of treatment for this patient?
A) Surgical correction between ages 2 and 5
B) Surgical correction between ages 6 and 12
C) Surgical correction between ages 13 and 17
D) Surgical correction at skeletal maturity
The correct response is Option B.
Pectus excavatum is the most common congenital chest wall deformity, occurring in approximately 1 in 400 live births. The condition is more common in males, and there is a positive family history in 30 to 40% of patients. The etiology is thought to be multifactorial and associated with increased incidence of congenital cardiac abnormalities, connective tissue disorders (e.g., Marfan and Ehlers-Danlos syndromes), and scoliosis. Treatment options have shifted from the traditional open technique involving sternal osteotomy and resection of abnormal costal cartilage to minimally invasive options such as the Nuss procedure and minimally invasive technique for repair of excavatum (MIRPE), which utilizes thoracoscopy and placement of intrathoracic retrosternal support bars to reposition the sternum and allow gradual remodeling over a period of 2 to 4 years. The ideal timing of repair is mid-adolescence, usually between ages 6 and 12.
A 56-year-old man is evaluated because of gynecomastia. Physical examination shows mild, diffuse breast enlargement with no visible inframammary fold or ptosis. Which of the following is the most appropriate surgical correction of this patient’s condition?
A) En bloc resection of skin and breast tissue with free nipple grafting
B) Open excision of breast tissue with mastopexy
C) Subcutaneous mastectomy with nipple preservation
D) Suction-assisted lipectomy
E) Superior periareolar excision with skin excision
The correct response is Option D.
The treatment of gynecomastia is based on the degree of breast enlargement and the extent of ptosis that is noted on examination. Grade 1 gynecomastia is minimal breast hypertrophy without ptosis. Grade II gynecomastia is moderate hypertrophy without ptosis. Grade III gynecomastia is severe hypertrophy with moderate ptosis. Grade IV gynecomastia is severe hypertrophy with severe ptosis. The treatment of mild to moderate gynecomastia without ptosis is suction-assisted lipectomy. Direct periareolar excision with skin excision and subcutaneous mastectomy are not indicated for gynecomastia without ptosis. Mastopexy and free nipple grafting techniques are indicated for gynecomastia with severe ptosis.
Which of the following is not a strong indication for skeletal reconstruction from chest wall resection?
A) two-rib segmental loss
B) resection of manubrium
C) resection of five ribs in an irradiated field
D) resection of body of sternum
E) altered pulmonary function following resection
Correct answer is option a.
Loss of the manubrium or body of the sternum will result in moderate to severe physiologic effects. More extensive skeletal resection may be tolerated in the irradiated thorax due to fibrosis following radiation, however, resection greater than 4 ribs may have significant physiologic effects. Pulmonary function tests before and after resection may help in the assessment of physiologic effects of the resection. Most authors agree that two-rib segmental loss can be reconstructed with soft tissue alone.
A 9-year-old boy is referred for evaluation because his breasts have been enlarging during the past six months without pain. Tanner stage is 1. Height and weight are in the 50th percentile. He takes no medications, and his medical history is noncontributory. Which of the following is the most appropriate next step in establishing the diagnosis?
A ) Abdominal CT and renal ultrasonography
B ) Karyotyping
C ) Measurement of serum gonadotropin concentrations
D ) Testicular examination and ultrasonography
E ) No further testing is necessary
The correct response is Option D.
Gynecomastia, defined as benign enlargement of the male breast, can affect as many as 65% of adolescent boys. It is a condition that reflects an elevated ratio of estrogens to androgens caused by age, disease, drugs, or idiopathic factors. Gynecomastia follows a triphasic distribution seen most frequently in the neonate, in early puberty, and with advanced age as a result of normal hormonal shifts that result in a relative preponderance of estrogens over androgens. For this reason, gynecomastia in these patients is considered physiologic and rarely requires workup beyond a careful history and physical examination.
The patient described comes to the office with early breast enlargement without other signs of pubertal development. Prepubertal gynecomastia is an uncommon condition, which should prompt a thorough workup for a possible pathologic cause. This is also true for patients of any age with bilateral idiopathic disease. While hormonal screening has previously been considered the first step in evaluating a patient with idiopathic gynecomastia, recent studies have suggested that these tests are rarely cost-effective. If hormone testing does suggest an occult testicular tumor, ultrasonography will be required for confirmation. Ultrasonography is therefore a preferred screening method, as it is cheaper and more accurate than hormone assays.
While specific workups have historically been suggested depending on age at presentation, the arbitrary use of screening radiologic or laboratory tests has not been found to be cost-effective. The presence of a feminizing tumor in a patient with idiopathic gynecomastia is extremely rare, affecting fewer than 3% of patients. Moreover, breast hypertrophy is an uncommon sign of testicular neoplasms, occurring in 30% of Leydig cell tumors and less than 10% of beta HCG-producing carcinomas. Feminizing adrenal tumors are even rarer. Adrenal imaging is therefore not warranted.
Mammogram and/or ultrasonography of the breast are reasonable choices if history and physical examination are suggestive of breast cancer. Breast cancers usually present as a hard, unilateral mass in an older patient. Karyotype testing would be warranted to rule out Klinefelter syndrome if feminizing characteristics are found on physical examination.
A 65-year-old man is prescribed leuprolide acetate for prostate cancer. Which of the following is the most likely effect the drug will have on this patient’s breasts?
A) Darkening of the nipple-areola complex
B) Decrease in size
C) Galactorrhea
D) Mastodynia
E) Petechiae
The correct response is Option D.
Leuprolide acetate (Lupron) is used in the treatment of certain cancers, including prostate, endometriosis, and precocious puberty. Because it is a synthetic gonadotropin-releasing hormone (GnRH), it acts as an agonist of pituitary GnRH receptors. Its ultimate effect is a lowering of estradiol and testosterone levels through downregulation of luteinizing hormone and follicle-stimulating hormone secretion.
Gynecomastia and breast tenderness are known side effects of leuprolide treatment for prostate cancer. Selective estrogen receptor modulators (SERMs), such as tamoxifen, are recommended for the prevention of gynecomastia as a result of antiandrogen monotherapy. Radiation as well has been shown to decrease the breast side effects of leuprolide acetate. Thus, an increase in size, rather than a decrease in size is expected. Darkening of the nipple areolar complex is often associated with pregnancy, but not leuprolide acetate. Galactorrhea is associated with drugs—commonly drugs that contain dopamine depleting agents, such as methyldopa, that cause receptor bock (such as Reglan), and that inhibit release, such as codeine and morphine. Histamine receptor blockade, from drugs such as cimetidine, can also cause galactorrhea. Pituitary tumors are also a cause of galactorrhea.
A thorough assessment of all medications and medical history is necessary for evaluation of a patient with gynecomastia and mastodynia.
A 1-day-old female newborn is evaluated because of a 4 × 4-cm defect after undergoing dural repair of a lumbar myelomeningocele. Which of the following is the most appropriate next step in management?
A) Negative pressure wound therapy
B) Split-thickness skin grafting
C) Coverage with a skin advancement flap
D) Coverage with a local musculofascial flap
E) Coverage with a free flap
Correct answer is option D.
The most appropriate next step in management is coverage with a local musculofascial flap. The major principle of myelomeningocele repair is to obtain a well-vascularized layer of soft-tissue coverage between the dural repair and skin repairs. The fascia overlying the paraspinous muscles can be turned over as flaps followed by paraspinous muscle advancement flaps to cover the underlying dural repair. This vascularized soft-tissue layer will minimize the risk of contact of cerebrospinal fluid with cutaneous bacteria and subsequent meningitis if either the dural repair or skin repair breaks down.
Vacuum-assisted wound therapy over a dural repair would increase the risk of cerebrospinal fluid leak, dural injury, and breakdown of the repair. A split-thickness skin graft over the dura would not adequately protect the spinal cord. Closing the skin directly over the dural repair would place the patient at risk for meningitis in the event of a cerebrospinal fluid leak, or if wound breakdown occurred along the incision line of the widely undermined skin flaps. The use of a free flap to cover the dural repair is unnecessary because local tissue (paraspinous muscles and fascia) is available. Harvesting the gluteal muscle(s) may cause significant donor site morbidity in a child already at risk for ambulatory problems from a neurologic deficit.
Gestationally during what weeks does the embryo undergo development of the trunk and torso?
A) 2-4 weeks
B) 1-4 weeks
C) 4-8 weeks
D) exactly 4 weeks
The correct answer is option C.
The understanding of human embryology has been developed from research in organisms such as Drosophila melanogaster, chick, and mouse. The development of the musculoskeletal trunk of the human body encompasses the union of skeletal and mesodermal tissues and is initiated by signals from surrounding tissues. This process occurs during the fourth through eighth weeks of gestation.
In bilateral component separation for abdominal wall reconstruction, which of the following points is most likely to be the area of greatest advancement?
A) Arcuate line
B) Ligament of Treitz
C) Suprapubic
D) Umbilicus
E) Xyphoid
The correct response is Option D.
Component separation for abdominal wall reconstruction involves release of the fascia lateral to the rectus abdominus muscles, just lateral to the semilunar line, dissecting the external oblique off the internal oblique muscles. This creates innervated musculofascial flaps that can be advanced medially for closure of ventral hernias. If the posterior rectus sheath is also dissected free, further advancements can be gained. Per rectus muscle, approximately 4 cm can be gained at the epigastric and suprapubic areas; 10 cm can be advanced at the waist. Therefore, the most advancement can be gained at the umbilicus, which is in the area of the waist. The xyphoid and ligament of Treitz are in the epigastric area, while the arcuate line is in the suprapubic area.
A 32-year-old woman is scheduled to undergo subtotal reconstruction of the vagina using a pedicled deep inferior epigastric artery perforator flap after tumor ablation. Which of the following is the most likely advantage of using this type of flap coverage rather than the pudendal thigh (Singapore) flap?
A) Better maintenance of sensory innervation
B) Less vaginal bulkiness
C) Less vaginal hair growth
D) More optimal mucus secretion
E) Single-staged procedure
The correct response is Option C.
Vaginal reconstruction for congenital vaginal agenesis or after tumor resection remains a challenging reconstructive endeavor. The ideal technique provides a vaginal canal with adequate dimensions, texture, appearance, and sexual function without excessive donor site morbidity.
The deep inferior epigastric artery perforator flap as a local regional flap has been described recently for vaginal reconstruction. It is a non-hair-bearing flap that is performed in a single stage but is insensate. Depending on the patient’s body habitus, it is less bulky than the musculocutaneous flaps (gracilis, vertical rectus) used for vaginal reconstruction, but more bulky than the Singapore flap. Two perforators are included in the flap to ensure viability. The major disadvantage is a conspicuous donor site scar.
The pudendal thigh flap, commonly referred to as the Singapore flap, is a local fasciocutaneous flap that has been widely used for neovaginal reconstruction. It is a thin, sensate flap based on the posterior labial arteries, which are a continuation of the perineal artery. Modifications of the flap have been described to enhance the viability and reach of the flap. It is a reliable, single-staged operation, but can result in endovaginal hair growth. This can be improved with preoperative electric or laser depilation of the vulvar portion of the flap.
Neither flap secretes mucus. Both are single-staged operations.
Which of the following findings is most indicative of Poland syndrome?
A) Abnormalities of the chest wall
B) Absence of the nipple
C) Absence of the sternal head of the pectoralis major muscle
D) Brachysyndactyly
E) Hypoplasia of the latissimus dorsi muscle
Correct answer is option C. ‘
For the definitive diagnosis of Poland syndrome, patients must have absence or hypoplasia of the sternal head of the pectoralis major muscle on the affected side. Other chest wall abnormalities can occur along with the absence of, or hypoplasia of, the ipsilateral breast. These include the latissimus dorsi and serratus anterior muscles. Brachysyndactyly of the ipsilateral upper extremity can be present as well. 
A 56-year-old woman who has type 2 diabetes mellitus is scheduled to undergo removal of prosthetic mesh that became infected after ventral hernia repair. Weight is 311 lb (141 kg); BMI is 53 kg/m2. Reconstruction with component separation technique and onlay grafting of reinforcing mesh is planned. Compared with polytetrafluoroethylene, acellular dermal matrix is preferable for this patient because of its ability to undergo which of the following processes?
A) Revascularization
B) Contraction
C) Encapsulation
D) Imbibition
E) Inosculation
Correct answer is option A.
Implant infection is a common clinical complication of abdominal hernia repair. Use of biologic mesh in contaminated cases has gained widespread acceptance. The ability of acellular dermal matrix (ADM) to revascularize is most likely the key to its successful application in contaminated scenarios. Revascularization subsequently helps clear bacteria. Contraction is a normal part of wound healing mediated by myofibroblasts, which are not present in polytetrafluoroethylene (PTFE) or ADM. Encapsulation is a foreign body reaction, whereby the organism forms a sterile scar around an alloplastic material. Imbibition is the process by which autogenous grafts perform initial metabolic exchange through diffusion. Inosculation is the first stage of neoangiogenesis, whereby immature vasculature between graft and recipient bed begin to organize in preparation for ingrowth. Neither process occurs in PTFE or ADM, as there is no existing vasculature in either product.
Incomplete involution of the mammary ridge during embryonic development is most likely to result in which of the following?
A) Amastia
B) Gynecomastia
C) Inverted nipple
D) Poland syndrome
E) Polymastia
The correct response is Option E.
The breast develops as the result of bilateral thickening of ectoderm along the milk line, or mammary ridge, from the axillary to the inguinal region. Mammary buds begin to develop as growths within the epidermis and invade the deeper mesenchyme. Much of the ridge disappears as the embryo develops as the result of apoptosis, except for the primary buds in the pectoral regions. Failure of regression of the mammary ridge can result in accessory breasts (polymastia) or accessory nipples (polythelia). Accessory breast tissue occurs in 1 to 2% of live births and commonly occurs in the axillae.
Amastia is the complete absence of the mammary gland. This occurs due to either the failure of the mammary ridge to develop or the complete involution of the mammary ridge.
Gynecomastia is defined as benign enlargement of the male breast. While pathologic cases can exist, it is most typically due to a normal response of the breast tissue to circulating levels of estrogen.
Inverted nipples are due to failure of the mesenchyme to proliferate above the level of the skin.
Poland syndrome can have the following components: hypoplasia of the breast and nipple, absence of the sternocostal portion of the pectoralis major muscle, absence of the pectoralis minor muscle, abnormalities of the chest wall, and anomalies of the upper extremity. Many etiologies have been hypothesized, with the most widely accepted being an interruption of the embryonic blood supply to the upper limb.
For the 17 years old patient depicted below, what is the most appropriate treatment option:
A. DIEP flap
B. Retromuscular TE placement
C. Custom made silicon implant
D. Bilateral prophylactic mastectomies and immediate DTI breast reconstruction
E. Wait until patient is 21 for left side mastopexy
Poland syndrome is a congenital condition characterized by
complete or partial pectoralis major muscle agenesis, breast and
nipple aplasia or hypoplasia, and chest wall and rib malformation. A
subset of patients has ipsilateral arm/hand maldevelopment,
including brachydactyly and possibly syndactyly.
Reconstruction with a deep inferior epigastric artery perforator free
(DIEP)flap is the best option from the choices. Because of the chest
wall abnormalities, the anatomy of the internal mammary artery
should be considered in selecting the recipient vessel. The internal
mammary artery may be abnormally small or course retrosternal,
and the thoracodorsal artery may need to be considered. The
images show a staged postoperative picture following DIEP and
before revision.
Although expansion of the skin may be a reasonable initial choice,
placement of a total submuscular tissue expander would be difficult
because of the aplasia of the sternocostal head of the pectoralis
major muscle and possibly the entire muscle. While a custom-made
silicone implant would be a good choice for this patient, bilateral
prophylactic mastectomy is not indicated for Poland syndrome in the
absence of breast cancer or genetic elevated risk for breast cancer,
such as BRCA. Earlier surgical management is preferred to improve
psychosocial development and waiting until age 21 years has no
advantage. Left-sided mastopexy alone would not address the
volume discrepancy between the right and left breast causing
asymmetry.
During fetal development, failure of the mammary pit to elevate above the skin level results in which of the following deformities?
A) Accessory nipple
B) Amastia
C) Amazia
D) Athelia
E) Nipple inversion
The correct response is Option E.
The congenitally inverted nipple is common and occurs in 4% of infants. It results from the failure of the mesenchyme to proliferate above the level of the skin.
Amastia is absence of the entire breast. Athelia is absence of the nipple. Amazia is absence of the mammary gland with an intact nipple and areola. Polythelia, or accessory nipple, results from failure of regression along the milk line.
Which of the following abdominal wall defects may not require surgical intervention because it may close on its own?
A) Umbilical hernia
B) Diaphragmatic hernia
C) Omphalocele
D) Prune belly syndrome
E) Gastrochisis
The correct answer is option a.
Umbilical hernias are common occurrences in children where the umbilical ring fails to close. Depending on the severity of the lesion, they may require surgical intervention. However, over time, small umbilical hernias are able to close on their own without the need for surgery. Omphalocele and gastrochisis are open abdominal wall defects that need to be surgically closed and will not close on its own. A diaphragmatic hernia is where the herniation of the abdominal viscera through a defect in the diaphragm requiring surgery. Prune belly syndrome is a defect where there is lack of development of the abdominal wall.
Which of the following posterior torso defects is most likely not covered by skin?
A) Myelocele
B) Meningocele
C) Myelomeningocele
D) Spina bifida occulta
E) Spinal teratoma
Correct answer is option C.
Myelomeningocele is a defect in which there is open dysplasia of the spinal cord and meninges. It is usually not covered with skin and requires immediate surgical intervention to prevent deterioration of neurologic function and risk for infection. Myelocele, meningocele, spina bifida occulta, and spinal teratoma are all usually covered by skin, although in some cases, the skin may be thin, pigmented, covered with hair or depressed.
A 10-year-old boy is brought to the office because of the findings shown in the photograph. Which of the following is the most likely diagnosis?
A) Amniotic band syndrome
B) Axillary web syndrome
C) Poland syndrome
D) Popliteal pterygium syndrome
E) Waardenburg syndrome
The correct response is Option C.
The patient has Poland syndrome with an unusual axillary web. The defining clinical feature is underdevelopment or absence of the sternal head of the pectoralis major muscles (seen in the photograph), but the deformity can lead to other anomalies such as absence of the nipple, the areola, or portions of the anterior chest wall. Cardiac anomalies may also be present. The ipsilateral extremity and hand are often smaller or hypoplastic, and this finding can be subtle or pronounced. The fingers are typically shorter and smaller than the contralateral side, and there may be webbing between the fingers. In severe forms, the central fingers are mere vestiges.
The other choices do not fit with this clinical photograph. Amniotic band syndrome usually leads to amputation or severe constriction of the hand or digits; this is not seen here. Waardenburg syndrome is a genetic condition that causes hearing loss and pigmentation changes to the hair, skin, and eyes. Popliteal pterygium syndrome is a condition associated with cleft lip/palate, webs of the popliteal space, and syndactyly of the toes or fingers. The axillary web seen here is not a described feature. Axillary web syndrome, or cording, is an acquired web that usually follows axillary node dissection in the treatment of breast cancer. This does not fit the clinical scenario.
A 63-year-old man comes to the office for consultation regarding correction of large breasts. Detailed history and physical examination show no cause of his condition. The breasts are a size C cup with ptosis and excessive skin. The nipples are 5 cm in diameter. Mastectomy and free nipple grafts are planned. The desired new size and shape of each areola are closest to which of the following?
(A) 1 cm, round
(B) 2 cm, round
(C) 2 cm, oval
(D) 3 cm, oval
(E) 4 cm, round
The correct response is Option D.
Larger forms of gynecomastia with significant ptosis present a challenge to plastic surgeons with respect to the size, shape, and position of the nipple on the chest wall. In addition, the nipple-areola complex may need to be reconstructed due to loss from cancer or trauma.
Two recent studies investigated the anatomical parameters of the nipple-areola complex in men. These studies demonstrated the following characteristics. More than 90% of the male subjects had nipples that were oval in configuration. The average areolar diameter in one study was 2.8 cm. The average areolar diameter in the other study was 2.7 cm. Furthermore, in men, the position of the nipple on the chest wall is typically 20 cm from the sternal notch and 18 cm from the midclavicular line. The ideal nipple-to-nipple distance in men is 21 cm.
Which of the following is the most appropriate term for the amount of air that is inspired and expired with a single breath during normal resting respiration?
A) Dead-space volume
B) Functional capacity
C) Residual volume
D) Tidal volume
E) Vital capacity
The correct response is Option D.
Tidal volume (VT) is the volume of air that is moved into or out of the lungs during quiet breathing. Tidal volume can be measured directly through spirometry or estimated based on a patient’s ideal body mass. It is a key parameter in mechanical ventilation to allow adequate ventilation without causing barotrauma to the lungs.
Vital capacity (VC) is the volume of air expired after deepest inspiration.
Functional capacity is a physiologic description of an individual’s ability to complete activities of daily living. It can be estimated through exercise treadmill testing and reported in metabolic equivalents (METs).
Residual volume (RV) is the volume of air remaining in the lungs after maximal exhalation.
Dead-space volume is the volume of air inhaled that does not take part in gas exchange. This volume can include both gas that remains in conducting airways (e.g., trachea, bronchi) during respiration and gas that reaches nonfunctional alveoli (e.g., nonperfused lung parenchyma following pulmonary embolism).
A 65-year-old man who underwent three-vessel coronary artery bypass grafting (CABG) five weeks ago comes to the office because he has a draining lesion near the sternotomy incision. The CABG procedure included harvest of the patient €™s left internal mammary artery. Temperature is 38.7EC (101.7EF). Physical examination shows a 3-mm papule at the manubrium of the healed sternotomy incision. CT scan of the chest shows a sinus tract leading to the internal sternal plate. In addition to removal of sternal wires and debridement of the wound, which of the following is the most appropriate management?
(A) Continuous irrigation
(B) Negative-pressure wound therapy
(C) Omental flap
(D) Pectoralis major muscle flaps
(E) Rectus abdominis muscle flaps
The correct response is Option D.
The most effective treatment of the sternal wound infection described is initial debridement to healthy bleeding tissue and bone, removal of all foreign bodies, and immediate closure with bilateral pectoralis major muscle flaps. Post €‘sternotomy wounds are classified according to the duration of time that has elapsed between the sternotomy and the clinical onset of the infection. Type I infections occur within the first week after sternotomy and have no bony involvement. Type II infections occur during the second to fourth weeks after sternotomy. Bony involvement is frequent, but chostochondritis is rare. Type III infections occur months to years after sternotomy and typically involve chronic draining sinus tracts. Osteomyelitis, chostochondritis, and retained foreign bodies are all common.
The patient described had a Type III infection with a deep sinus tract as confirmed by CT scan. Although delayed closure had previously been considered the treatment of choice for Type III sternotomy wounds, more recent data suggest that a single-stage approach with bilateral pectoralis major muscle flaps results in quicker recovery, improved cosmesis, and decreased morbidity and mortality compared with staged approaches. Given the chronicity of the wound, all foreign bodies, including retained suture material and wires, should be removed.
Pectoralis major turnover flaps provide additional bulk and might therefore be considered preferable. However, the left internal mammary artery has been harvested in this patient, compromising the blood supply to the left pectoralis major turnover flap.
An otherwise healthy 47-year-old man comes to the office because of the recurrent ventral hernia shown. He does not smoke cigarettes. Physical examination shows “Swiss cheese–type” defects, which are confirmed by CT scan. He desires reconstruction. Which of the following is the most appropriate surgical treatment?
A) Component separation with mesh placement
B) Laparoscopic hernia repair with mesh placement
C) Open hernia reduction with bridging mesh placement
D) Open reduction with free tensor fascia lata flap
E) Total autologous component separation
The correct response is Option A.
Given the size and history of the defect, the patient described has a significant chance of recurrence of his hernia. To optimize the chances of a functional recovery that is durable and has the lowest chance of recurrence, the component separation technique with mesh reinforcement (synthetic or biologic) is indicated. The component separation technique, originally described by Ramirez et al. in 1990, allows for recruitment of the rectus complex towards the midline to facilitate primary reapproximation (in the best-case scenario) or at least decrease the size of the residual defect. Studies have shown that reinforcement of hernia repairs with mesh decreases recurrence rates by 50 to 75%, even in secondary repairs. It is not clear, however, whether the mesh should be placed as an overlay or underlay. Nonetheless, mesh reinforcement leads to lower recurrence rates, especially if combined with component separation. Specifically, component separation allows for reduction of the hernia defect size after hernia reduction, and leads to lower recurrence rates versus hernia reduction and bridging mesh repair alone.
Laparoscopic repair is a useful technique in hernia repairs. However, the complex, large “Swiss cheese–type” defect described lends itself more to open repair in terms of facilitation of reduction, optimization of safety (avoidance of unrecognized bowel injury), and decrease in operative time.
A tensor fascia lata flap, originally described for large hernia repairs because it allows transfer of fascia to the abdomen, does not demonstrate superior results compared to available meshes. It also incurs donor site morbidity versus the mesh reinforcement techniques.
An 18-year-old woman comes to the office because of a small area of titanium extruding through the skin overlying the sacrum 12 months after undergoing a successful posterior spinal fusion and an autologous bone graft for myelodysplasia and lumbosacral kyphotic deformity. Physical examination shows a midline scar with a 2 × 2-cm wound at the level of the fourth lumbar vertebra with a visible screw. Which of the following is the most appropriate next step in management?
A) Bilateral skin advancement flaps
B) Free rectus abdominis muscle flap
C) Gluteus maximus muscle flap
D) Negative pressure wound therapy and skin graft
E) Removal of the hardware
The correct response is Option E.
The most appropriate next step in management of the exposed titanium is removal of the hardware. The titanium rod or one of the screws likely loosened and subsequently migrated to extrude through the skin. The vertebral column is adequately fused 1 year following the procedure. The orthopedics team should be consulted regarding removal of the hardware. Once the hardware is removed, the wound is easily repaired using delayed primary closure or by allowing it to heal secondarily.
Although local skin flaps, skin grafts, regional muscle transfer, and free flaps will provide vascularized soft-tissue coverage over the exposed hardware, these options ultimately will fail. The exposed hardware is much stronger than any soft-tissue coverage and will extrude through a flap, especially when the patient is supine. Definitive management requires eliminating the cause of the problem, which is malpositioned hardware, and not inadequate soft-tissue coverage.
A patient is born with a small, narrow chest and presents with respiratory distress. It was determined that the patient was born with asphyxiating thoracic dystrophy. Which of the following additional findings is most likely?
A) Absent clavicle
B) Omphalocele
C) Cleft lip
D) Athelia
E) Limb shortness
The correct answer is option E.
Asphyxiating thoracic dystrophy, Jeune syndrome, is an autosomal recessive disorder in which patients may present with alveolar hypoventilation as a result of short horizontally positioned ribs. Patients are also born with variable short and wide long bones of the extremities. Other findings include irregular costochondral junctions, elevated clavicles, short iliac bones, and hypoplastic phalanges of the hands and feet. Other complications that may develop later in life include renal, hepatic, and pancreatic. Associate position position rips
A 54-year-old man presents for elective repair of a large, recurrent ventral hernia. Plastic surgery is consulted because of the loss of abdominal domain. Posterior component separation with placement of biologic mesh is planned. Above the level of the arcuate line, which of the following structures come together to form the posterior rectus sheath?
A) External oblique aponeurosis and internal oblique aponeurosis
B) External oblique aponeurosis, internal oblique aponeurosis, and transversus abdominis aponeurosis
C) Internal oblique aponeurosis and transversus abdominis aponeurosis
D) Transversalis fascia and peritoneum
The correct response is Option C.
A comprehensive understanding of the anatomy of the anterior abdominal wall is critical when performing posterior component separation. Above the arcuate line, the anterior rectus sheath is derived from the external oblique aponeurosis and the anterior component of the internal oblique aponeurosis. The posterior rectus sheath is derived from the posterior component of the internal oblique aponeurosis and the aponeurosis of the transversus abdominis muscle. The rectus abdominis muscle sits between the anterior and posterior rectus sheaths superior to the arcuate line. Below the arcuate line, the anterior rectus sheath is derived from the aponeuroses of all three muscles: the external oblique, the internal oblique, and the transversus abdominis muscles. The posterior rectus sheath does not exist below the arcuate line. The rectus muscle is only separated from the abdominal viscera by the transversalis fascia and the peritoneum.
References
Match the blood supply for the trapezius muscles:
A) transverse cervical artery
B) thoracodorsal branch of the subscapular artery
C) pectoral branch of the thoracoacromial trunk
Correct cancer is option a
The trapezius muscle is supplied by the transverse cervical artery, innervated by the accessory nerve (motor) and cervical spinal nerves C3 and C4 (motor and sensory). It arises from the external occipital protuberance and the medial third of the superior nuchal line of the occipital bone (both in the back of the head), from the ligamentum nuchae, the spinous process of the seventh cervical, all the thoracic vertebrae and from the corresponding portion of the supraspinal ligament. It inserts on the posterior border of the lateral third of the clavicle, acromion process and spine of the scapula.
A 27-year-old woman is evaluated for a recurrent abdominal desmoid tumor. CT scan shows a mass that occupies the full-thickness right musculofascial abdominal wall, involving the rectus abdominis muscle and oblique muscles, including lateral to the semilunar line. Resection is performed. Photographs of the defect are shown. Which of the following is the most appropriate management?
A) Bilateral component separation, primary skin closure with incisional topical negative pressure wound therapy, adjuvant chemotherapy
B) Left component separation, bridging wide intraperitoneal underlay biologic mesh, primary closure skin
C) Pedicled right anterolateral thigh flap with rectus femoris, no mesh
D) Placement of a bridging inlay of biologic mesh with primary split-thickness skin grafting and negative pressure wound therapy
E) Placement of a bridging inlay of uncoated heavyweight polypropylene mesh with adjuvant radiation therapy
The correct response is Option B.
The lesion in this patient is a recurrent desmoid tumor, also known as aggressive fibromatosis. It is a benign tumor, usually found in younger patients between 10 and 40 years of age, and is locally aggressive. It is often associated with pregnancy and previous surgery and can frequently recur. Management is en bloc, full-thickness, wide local excision (usually with frozen section confirmation of negative margins). An aggressive full-thickness abdominal wall resection is standard of care, making reconstruction more challenging.
The more durable and functional reconstruction entails complete restoration of the abdominal wall, especially musculofascial components, in a primary reapproximation. However, depending on the size of the tumor and resultant defect, this may not be possible. Basic principles, after obtaining proper margins after resection of the tumor, would then be reduction in defect size to the maximal extent possible and wide bridging underlay of mesh with at least 4- to 5-cm margins in all directions. Bridging inlay, whereby a mesh is simply sewn to the margins of the defect, has clearly been shown to be inferior in terms of recurrence rates.
Mesh choice can be either synthetic or biologic, although if significant contamination exists, if soft-tissue coverage is tenuous, or if one desires to decrease the amount of adhesion formation when placing mesh directly against the bowel, one should consider placement of biologic mesh, accepting the fact that there is a higher incidence of postoperative bulges using these materials, by and large.
In this case, only a left component separation is possible, given that the tumor has invaded the right rectus muscle and obliques, precluding their use for myofascial advancement.
There is no role for chemotherapy or radiation therapy in the treatment of these tumors.
Coverage with a right anterolateral thigh flap, with or without rectus femoris, can reconstruct the soft-tissue defect, but avoiding the use of mesh in a defect over 4 cm has a significantly higher chance of a recurrent hernia and would not be standard of care.
Primary skin grafting on top of a nonvascularized thick piece of acellular dermal matrix will not “take,” even with use of negative pressure wound therapy.
A 21-year-old woman comes to the office because of a lump 2 cm below the inframammary fold. She says she has “always had it,” but it grew larger during pregnancy and has not decreased. The mass was painful during breast-feeding, and it is occasionally tender. Which of the following is the most likely diagnosis?
A) Epidermal inclusion cyst
B) Lipoma
C) Polymastia
D) Polythelia
E) Sarcoma
The correct response is Option C.
Patients often come to plastic surgeons with subcutaneous masses. An understanding of the differential diagnosis is helpful to counsel patients. Accessory breast tissue (polymastia) along the milk line is common. This breast tissue is responsive to hormonal influence, and patients will describe cyclical pain and swelling coinciding with menses and with milk letdown. Surgical excision can be performed. Polythelia is the presence of accessory nipples. Lipomas, epidermal inclusion cysts, and sarcomas can occur as subcutaneous masses, but they do not change in character based on hormonal influence.
A 59-year-old man is scheduled for reconstruction of a central abdominal wall hernia measuring 10 cm wide x 30 cm long. A surgical approach using posterior component separation is planned. Which of the following locations for fascial incision most accurately describes the technical considerations of this procedure?
A) Across the lateral intercostal neurovascular bundles
B) Along the mid-axillary line
C) Medial to the linea semilunaris
D) Parallel to the subcostal border
E) Vertical bisection of the rectus muscles
The correct response is Option C.
Component separation of the abdominal wall was initially described for the anterior components, that is, those which are located anterior to the rectus muscle. A more recent development involves component separation of the layers located posterior to the rectus fascia. The technique of posterior component separation begins with a vertical incision of the posterior rectus sheath 0.5 cm medial to the linea semilunaris and continues laterally in the avascular plane posterior to the transversalis muscle. It can extend as far posteriorly as the psoas muscle if needed. In the event that the posterior layer cannot be approximated in the midline, an interposition patch of omental fat, hernia sac, or absorbable mesh is used. Concurrent use of non-absorbable mesh to reconstruct deficient anterior layers may be used in conjunction with the posterior separation technique as long as the mesh is separated from viscera with an intact posterior layer. A benefit of the posterior separation technique is the preservation of the lateral neurovascular bundles preserving the dynamic function of the rectus muscles. The technique may be utilized even when fascial defects are not situated in the midline or are located adjacent to bony landmarks.
A 5-year-old male has a cerebrospinal fluid leak and a 3 x 3-cm area of wound dehiscence involving the posterior trunk following tethered cord repair. Which of the following is the most appropriate method to reconstruct the wound?
A) Gluteal muscle flap and skin advancement flap
B) Latissimus muscle turnover flap and skin advancement flap
C) Local fascial flap and skin advancement flap
D) Skin advancement flap
E) Split-thickness skin graft
Correct answer this option C.
The most appropriate method to reconstruct the wound is a local fascial flap and skin advancement flap. The major principle of tethered cord and myelomeningocele repair is to obtain a well-vascularized layer of soft tissue coverage between the dural and skin closures. The fascia overlying the paraspinous muscles can be turned over as flaps to cover the underlying dural repair. This vascularized soft tissue layer will minimize the risk of cerebrospinal fluid leak by reinforcing the dural repair. In addition, the fascial flaps will prevent contact with cutaneous bacteria and subsequent meningitis if either the dural repair or skin repair breaks down. A split-thickness skin graft over the dura would not adequately protect the spinal cord. Closing the skin directly over the dural repair using skin advancement flaps would place the child at risk for meningitis in the event of a cerebrospinal fluid leak or if wound breakdown occurred along the incision line of the widely undermined skin flaps. The use of a regional gluteal or latissimus muscle flap to cover the dural repair is unnecessary because local tissue (paraspinous muscle fascia) is available. Harvesting the gluteal or latissimus muscles also may cause significant donor site morbidity in a child already at risk for ambulatory problems from a neurological deficit.
A 45-year-old man undergoes abdominal reconstruction with a component separation technique. After release of the external oblique muscle bilaterally, which of the following best explains the preservation of flexion of the trunk?
A) Preservation of the nerves between the anterior rectus fascia and pyramidalis muscle
B) Preservation of the nerves between the external oblique and internal oblique muscles
C) Preservation of the nerves between the internal oblique and transversalis muscles
D) Preservation of the nerves between the internal oblique and transversus abdominis muscles
Correct answer is option D.
The rectus muscle is the primary flexor of the pelvis. Its segmental nerve supply enters the rectus sheath laterally in a plane between the internal oblique and transversus muscles. Separation of components is a versatile and powerful technique for autologous abdominal wall reconstruction. Along with the ability to close large midline ventral defects primarily, the rectus muscle remains innervated because only the aponeurosis of the external oblique is released. If correctly performed, the internal oblique layer will shield the underlying nerve supply to the rectus and result in a functional abdominal wall repair.
A 54-year-old man presents for elective repair of a large, recurrent ventral hernia. Plastic surgery is consulted because of the loss of abdominal domain. Posterior component separation with placement of biologic mesh is planned. Above the level of the arcuate line, which of the following structures come together to form the posterior rectus sheath?
A) External oblique aponeurosis and internal oblique aponeurosis
B) External oblique aponeurosis, internal oblique aponeurosis, and transversus abdominis aponeurosis
C) Internal oblique aponeurosis and transversus abdominis aponeurosis
D) Transversalis fascia and peritoneum
The correct response is Option C.
A comprehensive understanding of the anatomy of the anterior abdominal wall is critical when performing posterior component separation. Above the arcuate line, the anterior rectus sheath is derived from the external oblique aponeurosis and the anterior component of the internal oblique aponeurosis. The posterior rectus sheath is derived from the posterior component of the internal oblique aponeurosis and the aponeurosis of the transversus abdominis muscle. The rectus abdominis muscle sits between the anterior and posterior rectus sheaths superior to the arcuate line. Below the arcuate line, the anterior rectus sheath is derived from the aponeuroses of all three muscles: the external oblique, the internal oblique, and the transversus abdominis muscles. The posterior rectus sheath does not exist below the arcuate line. The rectus muscle is only separated from the abdominal viscera by the transversalis fascia and the peritoneum.
References
Use of botulin toxin in abdominal wall reconstruction has demonstrated improved:
A. Postoperative pain control
B. Hospital stay
C. Fascial closure
D. Wound breakdown
E. Surgical Site infection
Correct answer is option C.
Prehabilitation of the abdominal wall through botulinum toxin type A
(BTA) injections into the lateral abdominal wall musculature of
patients with large ventral hernias and loss of domain has been
demonstrated in multiple studies and meta-analyses to increase the
likelihood of primary fascial closure. Advocates for its use have used
varying types of BTA, doses, number of injections, and timing of
injections, but results have consistently shown that its use has
decreased the number of patients requiring bridging of the repair.
Based on the mechanism and peak effect of BTA, studies using the
injections at the time of repair have been less effective than their use
2 to 4 weeks prior to the planned operation.
Radiologic assessment of the patients at the time of injection and
immediately prior to surgery have demonstrated a lengthening of the
lateral abdominal wall musculature and decrease in the width of the
midline hernia. This lengthening and relaxation results in the
decreased excursion distance and improved compliance of the
abdominal wall, thus allowing for increased numbers of primary
fascial closures with or without component separation. Larger scale
studies are needed to determine other benefits of its use in this
population.
There are mixed results related to improved pain control, with some
studies reporting a reduction in oral morphine equivalents, while
others show no statistical difference. Improvement in complication
rates such as hospital stay, and recurrence are not consistent across
studies. The use of tissue expanders and progressive preoperative
pneumoperitoneum have also shown promise as adjuncts to
abdominal wall reconstruction. These have been used alone and in
conjunction with BTA injections with improved results regarding
achieving primary fascial closure. Since these are more invasive
procedures, they do carry unique risks (tissue expander infection/extrusion, intra-abdominal organ injury) that need to be
factored into the operative plan. The use of prehabilitation with BTA
has not been shown to decrease length of hospital stay or incidence
of wound breakdown, nor has it been shown to prolong ventilator
dependence or increase infection rate.
28-year-old man presents for consultation about the excess tissue of his chest. Medical history includes no weight change and no comorbidities. BMI is 26 kg/m2. Physical examination shows the nipple position is preserved in the craniocaudal dimension and firm soft tissue underlying the central breast bilaterally. Which of the following is the most appropriate surgical approach?
A) Horizontal scar breast reduction with nipple-areola grafting
B) Inverted-T pattern breast reduction with inferior pedicle
C) Liposuction alone
D) Periareolar access for open central glandular excision with liposuction
E) Skin-sparing mastectomy with nipple-areola tattoo
The correct response is Option D.
Scar minimization is a priority when possible in patients with gynecomastia. The patient’s acceptable BMI, absence of weight change, and physical description imply minimal skin excess, making him a candidate for limited scar techniques. Direct excision of the central gland through periareolar incision with liposuction for marginal contouring would address the patient’s condition without excess scarring. Liposuction alone is unlikely to adequately address the firm central glandular tissue under the nipple-areola complex. The other techniques such as inverted-T pattern breast reduction with inferior pedicle, horizontal scar breast reduction with nipple-areola grafting, and skin-sparing mastectomy with nipple-areola tattoo would all unnecessarily increase scarring.
A 20-year-old man is scheduled for gynecomastia reduction by the use of ultrasound-assisted lipoplasty. In planning this procedure, which of the following liposuction techniques has the highest risk of thermal injury to the skin?
(A) Dry
(B) Wet
(C) Superwet
(D) Tumescent
The correct response is Option A.
Ultrasound-assisted liposuction in a dry environment increases the risk of thermal injury and overlying skin necrosis. The introduction of subcutaneous fluid in the wet, superwet, and tumescent techniques helps cool the probe and decreases the risk of injury. Additionally, avoidance of end hits, strict continuous movement of the probe, continuous cold saline irrigation, and use of a probe sheath and a wet towel as a skin guard also protect against thermal injury when ultrasound energy is delivered to the tissues.
A 25-year-old man returns to the operating room for closure of the abdomen 2 weeks after undergoing small-bowel resection. The abdomen was left open after the resection and treated with abdominal negative pressure wound therapy. The bowel wall edema has improved, and the fascia and rectus muscles are 30 cm apart at the level of the umbilicus. There is no evidence of contamination. Abdominal wound closure is planned. There is adequate skin for primary closure. Which of the following methods is most appropriate for fascial closure?
A) Acellular dermal matrix and negative pressure wound therapy
B) Component separation and acellular dermal matrix interposition
C) Skin grafting and negative pressure wound therapy
D) Subcutaneous tissue expansion, staged closure of skin flaps
E) Submuscular tissue expansion, staged closure of fascial flaps
The correct response is Option B.
Abdominal wall reconstruction after severe trauma involves evaluation of the skin and fascia. First, the surgeon needs to determine if there is sufficient skin and subcutaneous tissue for primary closure. If there is insufficient skin, then tissue expanders, local tissue rearrangement, or distant flaps need to be considered. Second, if there is insufficient fascia, then component separation with primary fascial closure and mesh onlay or a mesh interposition are options for fascial closure. Since the fascial defect is 30 cm, it is unlikely that primary fascial closure can be achieved with component separation alone. An interposition of acellular dermal matrix is appropriate to bridge the fascial gap if primary fascial closure cannot be achieved.
An 18-year-old woman with right-sided Poland syndrome requests improvement in the appearance of her chest. Physical examination shows absence of an anterior axillary fold on the right side and a pectus excavatum deformity with an overlying hypoplastic right breast. The nipple-areola complex is small, lateral, and raised by about 3 cm compared with the left side. Which of the following thorax, breast, nipple-areola complex (TBN) classifications best characterizes this patient’s Poland syndrome deformity?
A) T1, B1, N1
B) T1, B2, N1
C) T2, B1, N2
D) T3, B2, N3
The correct response is Option C.
Poland syndrome is likely a multifactorial genetic syndrome related to the embryologic timing and development of the subclavian arch. Its unifying finding is absence of the pectoralis major muscle but is variable in other manifestations of underdevelopment of the chest wall, breast, and ipsilateral upper extremity. Poland syndrome is most often reported as more frequent in males, but some series show equal expression in males and females. Additionally, up to 10% of patients may have associated dextrocardia.
Having a way to describe or classify a deformation in an organized fashion is helpful in planning reconstruction, determining results and outcomes, and discussing cases with colleagues. The thorax, breast, nipple-areola complex (TBN) system was proposed and published by Romanini et al. to do just that. Since that publication, further research by the group based on the TBN classification has been published and others have suggested modifications to include the presence or absence of ipsilateral upper extremity anomalies.
Thorax
T1: absence of all or part of pectoralis
T2: T1 + pectus excavatum or carinatum
T3: T1 + rib aplasia (usually 3 and 4)
T4: T1 + T2 + T3
Breast
B1: hypoplastic breast
B2: breast aplasia (amastia)
Nipple-areola complex
N1: hypoplastic NAC less than 2 cm displaced
N2: hypoplastic NAC more than 2 cm displaced
N3: athelia
A 1-day-old female newborn is evaluated because of a 4 × 4-cm defect after undergoing dural repair of a lumbar myelomeningocele. Which of the following is the most appropriate next step in management?
A) Negative pressure wound therapy
B) Split-thickness skin grafting
C) Coverage with a skin advancement flap
D) Coverage with a local musculofascial flap
E) Coverage with a free flap
The correct response is Option D.
The most appropriate next step in management is coverage with a local musculofascial flap. The major principle of myelomeningocele repair is to obtain a well-vascularized layer of soft-tissue coverage between the dural repair and skin repairs. The fascia overlying the paraspinous muscles can be turned over as flaps followed by paraspinous muscle advancement flaps to cover the underlying dural repair. This vascularized soft-tissue layer will minimize the risk of contact of cerebrospinal fluid with cutaneous bacteria and subsequent meningitis if either the dural repair or skin repair breaks down.
Vacuum-assisted wound therapy over a dural repair would increase the risk of cerebrospinal fluid leak, dural injury, and breakdown of the repair. A split-thickness skin graft over the dura would not adequately protect the spinal cord. Closing the skin directly over the dural repair would place the patient at risk for meningitis in the event of a cerebrospinal fluid leak, or if wound breakdown occurred along the incision line of the widely undermined skin flaps. The use of a free flap to cover the dural repair is unnecessary because local tissue (paraspinous muscles and fascia) is available. Harvesting the gluteal muscle(s) may cause significant donor site morbidity in a child already at risk for ambulatory problems from a neurologic deficit.
A 58-year-old man comes to the office with recurrent rectal cancer four years after undergoing low anterior resection with adjuvant radiotherapy. Open abdominoperineal resection is planned. Which of the following closure methods will most reliably reduce the incidence of perineal wound complications?
A) Gracilis flap closure
B) Negative pressure wound therapy
C) Primary closure
D) Rectus abdominis flap closure
E) Split-thickness skin grafting
The correct response is Option D.
Several retrospective studies and one randomized trial have shown that when compared to primary closure, the rectus abdominis myocutaneous flap reduces wound healing complications after abdominoperineal resection (APR). Gluteal and gracilis flaps have been used for reconstruction after APR; however, the data supporting their use is not as robust. Split-thickness skin grafting and negative pressure wound therapy are inappropriate for reconstruction of the APR due to the size of the wounds and the risk of evisceration.
A 70-year-old man is evaluated because of chest wall incision drainage associated with leukocytosis, high fever, and blood cultures positive for Staphylococcus aureus 14 days after undergoing quintuple coronary artery bypass grafting using both internal mammary arteries. History includes type 2 diabetes mellitus, hypertension, hypercholesterolemia, and chronic obstructive pulmonary disease. He has smoked one pack of cigarettes daily for the past 40 years. BMI is 32 kg/m2. In addition to broad-spectrum antibiotic therapy, which of the following is the most appropriate initial management of this condition?
A) Debridement and negative pressure wound therapy
B) Debridement and reconstruction with a pectoralis turnover flap
C) Debridement and reconstruction with sternal plating
D) Debridement with primary rewiring and wound irrigation
Correct answer is option a.
 Post-sternotomy mediastinitis is a severe complication, with an incidence ranging between 0.2 and 8%. Risk factors include advanced age, diabetes mellitus, morbid obesity, reoperation for bleeding, and use of bilateral internal mammary arteries. Deep sternal wound infections are more serious and have a higher mortality rate than superficial or sterile sternal wounds. The most common organisms are Staphylococcus aureus and Staphylococcus epidermidis, but mixed gram-positive and gram-negative infections are not uncommon. Broad-spectrum antibiotic therapy to include coverage of MRSA infection and Pseudomonas should be instituted. Nevertheless, debridement is the mainstay of therapy. Adequate sternal and soft-tissue debridement is vital and can be combined with immediate sternal closure with delayed primary wound closure versus delayed closure of the entire wound with or without flaps to aid in obliterating any dead space. Many studies have demonstrated that the use of negative pressure wound therapy for wound coverage as a bridging method to final wound closure decreases the morbidity of these patients and is associated with decreased recurrent infection and treatment failure rates compared with conventional therapy, specifically primary rewiring or plating, especially for those at high risk. Flap closure at the time of sternal debridement can be performed; however, if bilateral internal mammary arteries are used for the bypass, a pectoralis turnover flap is not an option because of loss of the internal mammary artery for the pectoralis major. Options would include coverage with the pectoralis transposition flap based on the thoracoacromial artery, an omental flap, or a rectus abdominis flap based on the intercostal artery.
A 59-year-old man with hypertension, peripheral vascular disease, and coronary artery disease has sternal osteomyelitis after coronary artery bypass grafting with saphenous vein and left internal mammary artery grafts. Reconstruction with bilateral pectoralis advancement flaps is performed, but the flaps do not survive. Debridement is performed, and a defect remains. A photograph is shown. Which of the following is the most appropriate reconstruction?
A ) Adjacent tissue transfer and coverage with bilateral skin advancement flaps
B ) Coverage with left rectus abdominis turnover flap and skin grafting
C ) Coverage with left superior epigastric artery perforator fasciocutaneous flap
D ) Coverage with omental flap and skin grafting
E ) Coverage with right latissimus myocutaneous flap
The correct response is Option D.
The most appropriate option in the patient described is an omental flap with skin grafting. The defect encompasses the entirety of the sternum, from the sternal notch to the xiphoid. The first-line muscle flaps for this defect would usually be bilateral pectoralis muscle flaps (that do not include a left pectoralis turnover flap option due to the lack of a left internal mammary artery (IMA) used for the coronary graft). However, this option has already been used.
An omental flap has an axial blood supply that is not compromised (the gastroepiploic) and the ability to completely span the defect and obliterate the dead space; the skin graft provides the skin coverage needed to complete the reconstruction. Hence, it is the optimal choice in this patient.
An adjacent tissue transfer of random skin flaps bilaterally would not obliterate the dead space and would not provide robust axial blood supply to the necessary area, with the tenuous coverage existing along the sternal midline, exactly where the most vital blood supply would be necessary.
A left rectus abdominis turnover flap would not be an appropriate option for two reasons: 1) the left IMA has been harvested, thereby compromising the superior epigastric vessel on which this flap would be based; and 2) it would not reach the upper portion of the defect.
A left superior epigastric artery perforator fasciocutaneous flap would not be an appropriate option for two reasons: 1) the pedicle is compromised from the left IMA harvest; and 2) muscle flaps have been proven to be optimal for sternal defects more than fasciocutaneous flaps.
A right latissimus muscle flap may be a viable choice to get full coverage of the defect with dead space obliteration. However, the option indicates this is a myocutaneous flap, which would not allow for sufficient skin to be harvested to reconstruct the size of this defect.
A 2-year-old boy with a history of omphalocele presents for correction of a 5-cm abdominal bulge with a 3-cm widened scar over the bulge. Which of the following is the most appropriate next step in management of the bulge?
A) Anterior component separation
B) Interposition acellular dermal matrix placement
C) Interposition prosthetic mesh placement
D) Tensor fascia lata flap
E) Tissue expander placement
The correct response is Option A.
For most patients with omphaloceles less than 5 cm in diameter, a single operation involving a traditional anterior component separation from costal margin to iliac crest is sufficient to reduce the omphalocele and reapproximate the rectus diastasis. Extended component separations are typically only needed when the omphalocele is large and accompanied by a diaphragmatic hernia. Once the abdominal wall defect extends past 5 cm in diameter, a staged procedure involving the placement of tissue expanders and subsequent flap advancement must be considered. Autologous tissue options, such as the tensor fascia lata flap, may be needed for larger defects that can not be managed with component separation. While prosthetic or biologic mesh placement is used to correct large abdominal wall defects, it is typically used only after autologous options have failed or are not available.
A 19-year-old man with no available medical history presents for initial evaluation of gynecomastia. Which of the following is most appropriate to include in this patient’s workup?
A) Chest x-ray study
B) Hand x-ray study
C) Mammography
D) Testicular examination
The correct response is Option D.
Gynecomastia can occur because of imbalances of estrogen and testosterone, and it can be associated with obesity, certain drugs, and Klinefelter syndrome. It can also be associated with testicular tumors. Therefore, a testicular examination should be performed. Mammography is not necessary for a routine workup if physical examination shows no abnormal masses. A chest x-ray study is not needed for an otherwise healthy 19-year-old. An x-ray study of the hand can evaluate closure of growth plates when determining if puberty has completed. This is unlikely to be necessary for someone who is 19 years old.
A 45-year-old man undergoes abdominal reconstruction with a component separation technique. After release of the external oblique muscle bilaterally, which of the following best explains the preservation of flexion of the trunk?
A ) Preservation of the nerves between the anterior rectus fascia and pyramidalis muscle
B ) Preservation of the nerves between the external oblique and internal oblique muscles
C ) Preservation of the nerves between the internal oblique and transversalis muscles
D ) Preservation of the nerves between the transversus abdominis and the peritoneum
The correct response is Option C.
The rectus muscle is the primary flexor of the pelvis. Its segmental nerve supply enters the rectus sheath laterally in a plane between the internal oblique and transversalis muscles. Separation of components is a versatile and powerful technique for autologous abdominal wall reconstruction. Along with the ability to close large midline ventral defects primarily, the rectus muscle remains innervated because only the aponeurosis of the external oblique is released. If correctly performed, the internal oblique layer will shield the underlying nerve supply to the rectus and result in a functional abdominal wall repair.
A 39-year-old woman with exposed hardware after spinal fusion undergoes an external oblique turnover muscle flap procedure. The blood supply to this flap is primarily from which of the following vessels?
A) Deep circumflex iliac
B) Deep inferior epigastric
C) Iliolumbar
D) Intercostal
E) Subscapular
Correct answer is option D
The external oblique turnover muscle flap is not commonly used but has the potential to provide adequate coverage of large defects of the back that extend to the midline at the level of T10 to L4. The upper half of the flap is supplied by the 4th through 11th intercostal arteries, whereas the lower half is supplied by one or two vessels from the deep circumflex iliac artery (95%) or the iliolumbar artery (5%). The upper half has a strictly segmental blood and nerve supply, whereas the lower half has segmental innervation but derives its blood supply from one artery. 
Which of the following findings is most indicative of Poland syndrome?
A) Abnormalities of the chest wall
B) Absence of the nipple
C) Absence of the sternal head of the pectoralis major muscle
D) Brachysyndactyly
E) Hypoplasia of the latissimus dorsi muscle
Correct answer is option C.
For the definitive diagnosis of Poland syndrome, patients must have absence or hypoplasia of the sternal head of the pectoralis major muscle on the affected side. Other chest wall abnormalities can occur along with the absence of, or hypoplasia of, the ipsilateral breast. These include the latissimus dorsi and serratus anterior muscles. Brachysyndactyly of the ipsilateral upper extremity can be present as well.
A 57-year-old man comes to the office for consultation regarding enlargement of the breasts. Physical examination shows bilateral large, ptotic, female-appearing breasts with firm, tender, glandular-like tissue deep to each nipple. Laboratory studies show increased beta-human chorionic gonadotropin level. Which of the following studies is the most appropriate next step in establishing the diagnosis?
(A) Biopsy of the breast
(B) CT scan of the abdomen
(C) Mammography
(D) MRI of the brain
(E) Ultrasonography of the testes
The correct response is Option E.
In adults with gynecomastia, thorough medical evaluation is required. History and physical examination should identify new medications, drug and alcohol abuse, and endocrine, hepatic, or pulmonary disease. Laboratory studies should measure electrolytes, blood urea nitrogen, creatinine, testosterone (total and free), estradiol, follicle-stimulating hormone, luteinizing hormone, beta-human chorionic gonadotropin (_-hCG), prolactin, liver function, and thyroid function. Radiography should also be performed. These tests are intended to rule out germ cell tumors, primary hypogonadism, hyperthyroidism, androgen resistance, pituitary tumors, secondary hypogonadism, and lung cancer. Mammography is not routinely used unless there is a finding on physical examination that shows possible presence of breast cancer; there is no known association between gynecomastia and breast cancer (except in Klinefelter syndrome). Imaging of the brain is not routinely ordered unless there is some other finding suggestive of a brain tumor. If the _-hCG concentration is increased, ultrasonography of the testes is indicated to rule out germ cell and non–germ cell tumors. CT scan of the abdomen should be ordered only if ultrasonography of the testes is negative.
A 70-year-old man is evaluated because of chest wall incision drainage associated with leukocytosis, high fever, and blood cultures positive for Staphylococcus aureus 14 days after undergoing quintuple coronary artery bypass grafting using both internal mammary arteries. History includes type 2 diabetes mellitus, hypertension, hypercholesterolemia, and chronic obstructive pulmonary disease. He has smoked one pack of cigarettes daily for the past 40 years. BMI is 32 kg/m2. In addition to broad-spectrum antibiotic therapy, which of the following is the most appropriate initial management of this condition?
A) Debridement and negative pressure wound therapy
B) Debridement and reconstruction with a pectoralis turnover flap
C) Debridement and reconstruction with sternal plating
D) Debridement with primary rewiring and wound irrigation
The correct response is Option A.
Post-sternotomy mediastinitis is a severe complication, with an incidence ranging between 0.2 and 8%. Risk factors include advanced age, diabetes mellitus, morbid obesity, reoperation for bleeding, and use of bilateral internal mammary arteries. Deep sternal wound infections are more serious and have a higher mortality rate than superficial or sterile sternal wounds. The most common organisms are Staphylococcus aureus and Staphylococcus epidermidis, but mixed gram-positive and gram-negative infections are not uncommon. Broad-spectrum antibiotic therapy to include coverage of MRSA infection and Pseudomonas should be instituted. Nevertheless, debridement is the mainstay of therapy.
Adequate sternal and soft-tissue debridement is vital and can be combined with immediate sternal closure with delayed primary wound closure versus delayed closure of the entire wound with or without flaps to aid in obliterating any dead space. Many studies have demonstrated that the use of negative pressure wound therapy for wound coverage as a bridging method to final wound closure decreases the morbidity of these patients and is associated with decreased recurrent infection and treatment failure rates compared with conventional therapy, specifically primary rewiring or plating, especially for those at high risk.
Flap closure at the time of sternal debridement can be performed; however, if bilateral internal mammary arteries are used for the bypass, a pectoralis turnover flap is not an option because of loss of the internal mammary artery for the pectoralis major. Options would include coverage with the pectoralis transposition flap based on the thoracoacromial artery, an omental flap, or a rectus abdominis flap based on the intercostal artery.
A 32-year-old woman comes to the office for consultation regarding gender reassignment surgery. She is in a stable relationship with a supportive partner and has the support of her family. She is interested in mastectomy, hysterectomy, salpingo-oophorectomy, and phalloplasty. Referral to which of the following is the most appropriate next step in management?
A) Breast surgeon for mastectomy
B) Endocrinologist for hormonal manipulation
C) Gynecologist for hysterectomy and salpingo-oophorectomy
D) Psychiatrist for evaluation
E) Urologist to assist with phalloplasty
The correct response is Option D.
The most appropriate next step in management is referral to a psychiatrist for evaluation. Gender identity disorder, formerly known as transsexualism, should ideally be managed by a multidisciplinary team. Ideal management includes psychotherapeutic, hormonal, and surgical treatments. Initial care involves a diagnostic phase in two stages. The goal of the first stage is to establish the diagnosis according to well-defined criteria and to rule out other psychiatric disorders. The second stage of the diagnostic phase is a ?real-life test,? in which the patient assumes the role of the opposite sex. Ongoing supportive psychotherapy is often required. Some centers begin hormonal therapy during the ?real-life test? to assist the transition and to make the test more realistic. Most centers defer hormonal therapy until 1 year following initial contact with the patient; surgery is deferred for 2 years following contact. There are no well-established protocols to guide the surgical sequence.
Absence of which of the following structures is most characteristic of patients with Poland syndrome?
A) Clavicular head of the pectoralis major muscle
B) Ipsilateral ribs
C) Pectoralis minor muscle
D) Serratus anterior muscle
E) Sternocostal head of the pectoralis major muscle
The correct response is Option E.
What follows is the description from the original records found in Paris, France: “In 1841, Alfred Poland, a 19-year-old medical student at Guy’s Hospital in London, England, published the classic description of the syndrome that bears his name. Serving as an anatomy demonstrator, Poland recorded that, in his subject: ‘the whole of the sternal and costal portions of the pectoralis-major muscle were deficient; but its clavicular origin quite normal. In the left hand, the middle phalanges were absent. The web between the fingers extended to the first articulation the hand was shorter than the right the left thumb was quite normal.’ In a footnote, he indicated that ‘the hand has been deposited in the Museum of Guy’s Hospital.’ Poland was a popular teacher and later pursued a distinguished surgical career.” (Charlier P, Deo S, Galassi FM, Benmoussa N. Poland syndrome before Alfred Poland: the oldest medical description [Paris, France, 1803]. Surg Radiol Anat. 2019;41[10]: 1117-1118.)
Today, Poland syndrome presents a spectrum of chest wall anomalies ranging from simple to complex. These deficiencies are largely cosmetic, with the most common (simple) form presenting as a unilateral absence of the sternocostal head of the pectoralis major muscle. The deformity can also be complex, with ipsilateral absence of ribs, axillary webbing, and foreshortening of the hemithorax. Reconstructive options for the chest wall depend on anatomical severity, gender, associated anomalies, and, of course, the patient’s preference.
Clavicular head of pectoralis major muscle is generally present in these patients. Pectoralis minor, absence of serratus anterior, and rib absence may or may not be present in patients with Poland syndrome.
A 52-year-old woman with cancer of the right breast undergoes mastectomy and axillary node dissection, complicated by mastectomy flap necrosis requiring skin grafting. She completes adjuvant chemoradiation. One year later, she comes to the office with a fungating mass growing through the skin graft. Imaging demonstrates involvement of the fourth and fifth ribs with an anticipated skeletal defect of 4 × 4 cm. A photograph is shown. Which of the following is the most appropriate treatment for the skeletal reconstruction?
A) High-density porous polyethylene
B) Methyl methacrylate with mesh
C) 2.4-mm Titanium plate
D) No skeletal reconstruction
E) Vascularized rib
The correct response is Option D.
The principles of management of this recurrent right breast cancer include radical resection of all involved tissues (including ribs) and reconstruction with well vascularized flaps. In this case, a right latissimus muscle flap and skin graft was used for reconstruction. No alloplastic material was placed or skeletal thoracic cage reconstruction performed. This is common in these types of patients, because excessive fibrosis caused by the radiation to the chest wall prevents loss of respiratory efficiency through paradoxical motion which otherwise occurs in patients who have more than four ribs involved or a defect larger than 5 cm.
A 67-year-old woman with a large ventral hernia after prior exploratory laparotomy for trauma is scheduled for complex abdominal hernia repair with posterior components separation and mesh. The planned surgical technique involves division of which of the following muscles and locations?
A) External abdominal oblique lateral to the linea semilunaris
B) External abdominal oblique medial to the linea semilunaris
C) Internal abdominal oblique lateral to the linea semilunaris
D) Transversus abdominis lateral to the linea semilunaris
E) Transversus abdominis medial to the linea semilunaris
The correct response is Option E.
The posterior components separation technique involves dissection in the retro-rectus plane to release the transversus abdominis muscle medial to the linea semilunaris, allowing medialization of the posterior rectus sheath-transversalis fascia complex. This fascial layer is approximated to form the posterior layer of closure. The anterior layer, consisting of abdominal wall musculature, is approximated at the linea alba. Mesh is usually placed in between these two layers.
In the more commonly performed anterior components separation technique, the external abdominal oblique is released lateral to the linea semilunaris to permit medialization of the rectus abdominis muscle for abdominal closure. The internal abdominal oblique is not released in anterior component separation. The internal abdominal oblique fascia is released medial to the semilunaris in posterior component separation.
References
A 22-year-old man comes to the office reporting breast enlargement that began 7 years ago. Physical examination shows bilateral breast enlargement. Testicular examination is normal. BMI is 27 kg/m2. Which of the following evaluations is best for distinguishing gynecomastia from pseudogynecomastia in this patient?
A) History of medication use
B) Laboratory testing
C) Mammography
D) MRI
E) Physical examination
The correct response is Option E.
Gynecomastia is benign enlargement of the glandular breast tissue in a man. It is often related to factors that either results in an increase in estrogen production, or a decrease in androgen production. This can be due to the transient imbalances during puberty, or secondary to medication use, drug use (such as marijuana), testicular pathology, thyroid disease, liver disease, breast cancer, etc.
This is in contrast to pseudogynecomastia, which is enlargement of the breast due to fat deposition in the absence of glandular hypertrophy. Pseudogynecomastia is typically seen in the patient who is overweight or obese.
Pseudogynecomastia is distinguished from gynecomastia by physical examination. In true gynecomastia, one will palpate enlarged firm glandular breast tissue, as opposed to in pseudogynecomastia, where palpation will reveal a soft fatty breast throughout with no enlargement of the subareolar tissue.
History may suggest a pathologic etiology that will be confirmed on laboratory testing. Medication and drug use questioning will show if the gynecomastia is pharmacologic in origin. Mammography and MRI will aid in determining if there is a malignant etiology causing the breast tissue enlargement. However, it is not standard for gynecomastia evaluation.